Added lattice handout
This commit is contained in:
parent
fbce34cf82
commit
8774a3ba59
26
Advanced/Lattices/main.tex
Executable file
26
Advanced/Lattices/main.tex
Executable file
@ -0,0 +1,26 @@
|
||||
% use [nosolutions] flag to hide solutions.
|
||||
% use [solutions] flag to show solutions.
|
||||
\documentclass[
|
||||
solutions,
|
||||
singlenumbering
|
||||
]{../../resources/ormc_handout}
|
||||
|
||||
%\usepackage{lua-visual-debug}
|
||||
\renewcommand{\arraystretch}{1.2}
|
||||
\begin{document}
|
||||
|
||||
\maketitle
|
||||
<Advanced 2>
|
||||
<Spring 2023>
|
||||
{Lattices}
|
||||
{
|
||||
Prepared by Mark on \today \\
|
||||
}
|
||||
|
||||
|
||||
\input{parts/0 intro.tex}
|
||||
\input{parts/1 minkowski.tex}
|
||||
\input{parts/2 orchard.tex}
|
||||
|
||||
|
||||
\end{document}
|
BIN
Advanced/Lattices/orchard.png
Normal file
BIN
Advanced/Lattices/orchard.png
Normal file
Binary file not shown.
After Width: | Height: | Size: 116 KiB |
49
Advanced/Lattices/parts/0 intro.tex
Normal file
49
Advanced/Lattices/parts/0 intro.tex
Normal file
@ -0,0 +1,49 @@
|
||||
\definition{}
|
||||
The \textit{integer lattice} $\mathbb{Z}^n \subset \mathbb{R}^n$ is the set of points with integer coordinates. We call each point in the lattice a \textit{lattice point}.
|
||||
|
||||
\problem{}
|
||||
Draw $\mathbb{Z}^2$.
|
||||
|
||||
\vfill
|
||||
|
||||
|
||||
\definition{}
|
||||
We say a set of vectors $\{v_1, v_2, ..., v_n\}$ \textit{generates} $\mathbb{Z}^n$ if every lattice point can be written uniquely as
|
||||
$$
|
||||
a_1v_1 + a_2v_2 + ... a_nv_n
|
||||
$$
|
||||
for integer coeficcients $a_i$.
|
||||
|
||||
\problem{}
|
||||
Which of the following generate $\mathbb{Z}^3$?
|
||||
\begin{itemize}
|
||||
\item $\{ (1,2), (2,1) \}$
|
||||
\item $\{ (1,0), (0,2) \}$
|
||||
\item $\{ (1,1), (1,0), (0,1) \}$
|
||||
\end{itemize}
|
||||
|
||||
\vfill
|
||||
|
||||
\problem{}
|
||||
Find a set of vectors that generates $\mathbb{Z}^2$.
|
||||
|
||||
\vfill
|
||||
|
||||
\problem{}
|
||||
Find a set of vectors that generates $\mathbb{Z}^n$.
|
||||
|
||||
|
||||
|
||||
\vfill
|
||||
\pagebreak
|
||||
|
||||
\problem{}
|
||||
A \textit{fundamental region} of a lattice is the parallelepiped spanned by a generating set. The exact shape of this region depends on the generating set we use.
|
||||
|
||||
\vfill
|
||||
|
||||
\problem{}
|
||||
Draw two fundamental reions of $\mathbb{Z}^2$ using two different generating sets. Verify that their volumes are the same.
|
||||
|
||||
\vfill
|
||||
\pagebreak
|
104
Advanced/Lattices/parts/1 minkowski.tex
Normal file
104
Advanced/Lattices/parts/1 minkowski.tex
Normal file
@ -0,0 +1,104 @@
|
||||
\section{Minkowski's Theorem}
|
||||
|
||||
\theorem{Blichfeldt's theorem}
|
||||
Let $X$ be a finite connected region. If the volume of $X$ is greater than $1$, $X$ must contain two distinct points that differ by an element of $\mathbb{Z}^n$. In other words, there exist distinct $x, y \in X$ so that $x - y \in \mathbb{Z}^n$.
|
||||
|
||||
\problem{}
|
||||
Draw a region in $\mathbb{R}^2$ with volume greater than 1 that contains no lattice points. Find two points in that region which differ by an integer vector.
|
||||
\hint{Area is two-dimensional volume.}
|
||||
|
||||
\vfill
|
||||
|
||||
|
||||
\problem{Proof in $\mathbb{Z}^2$}
|
||||
The following picture gives the idea for the proof of Blichfeldt's theorem. Explain the picture and complete the proof.
|
||||
|
||||
\begin{center}
|
||||
\includegraphics[angle=90,width=0.5\linewidth]{proof.png}
|
||||
\end{center}
|
||||
|
||||
\begin{solution}
|
||||
The fundamental region of $\mathbb{Z}^2$ tiles the plane. Translate these tiles by lattice vectors to stack them on the fundamental region. Then since the union of the intersections of X with these tiles has area greater 1 and they are stacked on a region of area 1, there must be an overlap by a generalization of the pigeonhole principle (if there were no overlap then the sum of the areas would be less than or equal to 1). Take points $x, y$ in the overlap. Then $x - y$ is a lattice point corresponding to the difference in translates, which were lattice points. Hence, $x - y \in \mathbb{Z}^2$.
|
||||
\end{solution}
|
||||
|
||||
\vfill
|
||||
\pagebreak
|
||||
|
||||
|
||||
%\problem{}
|
||||
%Does your proof of Blichfeldt's theorem in $\mathbb{Z}^2$ extend to a proof of Blichfeldt's theorem in $\mathbb{Z}^n$?
|
||||
%\vfill
|
||||
|
||||
|
||||
\problem{}
|
||||
Let $X$ be a region $X$ of volume $k$. How many integral points must $X$ contain after a translation?
|
||||
|
||||
\vfill
|
||||
|
||||
\definition{}
|
||||
A region $X$ is \textit{convex} if the line segment connecting any two points in $X$ lies entirely in $X$.
|
||||
|
||||
\problem{}
|
||||
\begin{itemize}
|
||||
\item Draw a convex region in the plane.
|
||||
\item Draw a region that is not convex.
|
||||
\end{itemize}
|
||||
|
||||
\vfill
|
||||
\pagebreak
|
||||
|
||||
|
||||
|
||||
\definition{}
|
||||
We say a region is \textit{symmetric with respect to the origin} if for all points $x$ in the region, $-x$ is also in $X$.
|
||||
|
||||
\problem{}
|
||||
\begin{itemize}
|
||||
\item Draw a symmetric region.
|
||||
\item Draw an asymmetric region.
|
||||
\end{itemize}
|
||||
|
||||
\vfill
|
||||
|
||||
\theorem{Minkowski's Theorem}<mink>
|
||||
Every convex set in $\mathbb{R}^n$ that is symmetric with respect to the origin and which has a volume greater than $2^n$ contains an integral point that isn't zero.
|
||||
|
||||
|
||||
\problem{}
|
||||
Draw a few sets that satisfy \ref{mink} in $\mathbb{R}^2$. Which is the simplest region that has the properties listed above?
|
||||
|
||||
\vfill
|
||||
|
||||
|
||||
\problem{}
|
||||
Let $K$ be a region in $\mathbb{R}^2$ satisfying \ref{mink}. Scale this region by $\frac{1}{2}$, called $K' = \frac{1}{2}K$.
|
||||
|
||||
\begin{itemize}
|
||||
\item How does the volume of $K'$ compare to $K$?
|
||||
\item Show that the sum of any two points in $K'$ lies in $K$
|
||||
\item Apply Blichfeldt's theorem to $K'$ to prove Minkowski's theorem in $\mathbb{R}^2$.
|
||||
\end{itemize}
|
||||
|
||||
\vfill
|
||||
|
||||
\problem{}
|
||||
Let $K$ be a region in $\mathbb{R}^n$ satisfying \ref{mink}. Scale this region by $\frac{1}{2}$, called $K' = \frac{1}{2}K$.
|
||||
|
||||
\begin{itemize}
|
||||
\item How does the volume of $K'$ compare to $K$?
|
||||
\item Show that the sum of any two points in $K'$ lies in $K$
|
||||
\item Apply Blichfeldt's theorem to $K'$ to prove Minkowski's theorem.
|
||||
\end{itemize}
|
||||
|
||||
\begin{solution}
|
||||
\begin{itemize}
|
||||
\item The volume of $K'$ is $\frac{1}{2^n}$ the volume of $K$.
|
||||
\item Take $x, y \in K'$. It follows that $2x, 2y \in K$. Since $K$ is convex, we have that the midpoint of the line segment between $2x$ and $2y$ is in $K$, and so $\frac{2x + 2y}{2} = x + y \in K$.
|
||||
\item Since the volume of $K$ is greater than $2^n$, we have the volume of $K'$ is greater than one.
|
||||
Applying Blichfeldt's theorem, we can find two distinct points $x, y \in K'$ such that $x - y \in \mathbb{Z}^n$. Since $K'$ is symmetric with respect to the origin, we have that $-y \in K'$. Therefore, $x + (-y) \in K$ by the previous part. $x \neq y, x - y \neq 0$, so we have found a nontrivial integer point in $K$.
|
||||
|
||||
\end{itemize}
|
||||
\end{solution}
|
||||
|
||||
\vfill
|
||||
\pagebreak
|
44
Advanced/Lattices/parts/2 orchard.tex
Normal file
44
Advanced/Lattices/parts/2 orchard.tex
Normal file
@ -0,0 +1,44 @@
|
||||
\section{Polya's Orchard Problem}
|
||||
|
||||
You are standing in the center of a circular orchard of integer radius R. A tree was planted each integer lattice point, and each has grown to the same radius $r$. If the radius is small enough, you will have a clear line of sight through the orchard in certain directions. If the radius is too large, there is no line of sight through the orchard in any direction. See the figure below:
|
||||
|
||||
\begin{center}
|
||||
\includegraphics[width=0.5\linewidth]{orchard.png}
|
||||
\end{center}
|
||||
|
||||
|
||||
\problem{}
|
||||
Show that if $r < \frac{1}{\sqrt{R^2 + 1}}$, you have at least one directon with a clear line of sight. \\
|
||||
\hint{Take a look at the ray through the point $(R, 1)$ and calculate the distance from the closest integer points to the ray.}
|
||||
|
||||
\begin{solution}
|
||||
Consider the ray from the origin through the point $(R, 1)$. Clearly, the two closest lattice points are $(1, 0)$ and $(R − 1, 1)$. They are equally far from the ray so let's calculate the distance from $(1, 0)$ to our ray. Call this distance $\delta$. Consider the triangle with vertices $(0, 0)$, $(1, 0)$, and $(R, 1)$. Then the area of this triangle is $12$. On the other hand, the area is also given by $\frac{1}{2} \delta \sqrt{R^2 + 1}$. So, $\delta = \frac{1}{\sqrt{R^2+1}}$. Therefore, if $r < \frac{1}{\sqrt{R^2+1}}$, we will have a clear line of sight given by this ray.
|
||||
\end{solution}
|
||||
|
||||
|
||||
\vfill
|
||||
\pagebreak
|
||||
|
||||
\problem{}
|
||||
Show that there is no line of sight through the orchard if $r > \frac{1}{R}$. You may want to use the following steps:
|
||||
\begin{itemize}
|
||||
\item Show that there is no line of sight if $r \geq 1$.
|
||||
\item Suppose $r < 1$ and $r > \frac{1}{R}$. Then, $R \geq 2$. Choose a potential line of sight passing through an arbitrary point $P$ on the circle. Thicken this line of sight equally on both sides into a rectangle of width $2r$ tangent to $P$ and $-P$. From here, use Minkowski's theorem to get a contradiction. Don't forget
|
||||
to rule out any lattice points that sit outside the orchard but inside the rectangle.
|
||||
\end{itemize}
|
||||
|
||||
% Any interest in counting the number of trees in the orchard? If so, google the Gauss circle problem. If orchards are not your slice of fruit, maybe rational approximations are your cup of tea
|
||||
|
||||
\begin{solution}
|
||||
Suppose $r < 1$ and let $L$ be a potential line of sight. Consider the rectangle of width $2r$ tangent to $P$ and $-P$. Then this is convex and symmetric with respect to the origin. Its area is $(2R)(2r) > 4 \frac{R}{R} = 4$. By Minkowski, we have a nonzero integral point in this rectangle. Suppose first that the integer point is within the orchard. Then this means that there is a tree whose distance to the line is at most $r$. Therefore, this tree blocks the line of sight. Now notice that there is a part of this rectangle that sits outside the orchard. Can the integer point be in this region? This would mean its distance to the origin, $D$, would satisfy $D > R$. Now since this point is within a distance of $r$ of our line $L$, we have that $D < \sqrt{R^2 + r^2} < \sqrt{R^2 + 1}$. So we have that $R < D < \sqrt{R^2 + 1}$. Then $R^2 < D^2 < R^2 + 1$, but $D^2$ is an integer so this is impossible.
|
||||
\end{solution}
|
||||
|
||||
\vfill
|
||||
|
||||
\problem{Challenge}
|
||||
Prove that there exists a rational approximation of $\sqrt{3}$ within $10^{-3}$ with denominator at most $501$. Come up with an upper bound for the smallest denominator of a $\epsilon$-close rational approximation of any irrational number $\alpha > 0$. Your bound can have some dependence on $\alpha$ and should get smaller as $\alpha$ gets larger. \\
|
||||
\hint{Use the orchard.}
|
||||
|
||||
\vfill
|
||||
\pagebreak
|
||||
|
BIN
Advanced/Lattices/proof.png
Executable file
BIN
Advanced/Lattices/proof.png
Executable file
Binary file not shown.
After Width: | Height: | Size: 201 KiB |
Loading…
x
Reference in New Issue
Block a user